2
$\begingroup$

Consider the following random variable $$ Z=\min_i\{X_i+Y_i\} $$ for $-n\leq i\leq n$, where $X_i\overset{\mathrm{iid}}{\sim}\text{Exp}(\lambda)$, $Y_i\overset{\mathrm{iid}}{\sim}\text{Erlang}(|i|,\gamma)$, and $X_i \perp Y_i,\forall i$.

One can think of $Z$ as the time of first arrival (at $i=0$) of incoming waves starting from different locations ($i$). For each $i$, we assume it takes time $X_i$ to initiate a wave at point $i$ and time $Y_i$ to for the wave to reach $i=0$. $Z$ is the minimum of all possible times.

What can we say about $\mathbb{E}[Z]$?

Some ideas: In the case where we take $Y_i$ as its expected value, that is, $\mathbb{E}[Y_i]=|i|/\gamma$, we may actually compute a closed formula for $\mathbb{E}[Z]$, assuming non-negative time, $$\tag{1} \mathbb{E}[Z]=\mathbb{E}[\min_i\{X_i+|i|/\gamma\}]=\int_0^\infty \prod_i \min \{ 1,\exp(-\lambda(t-|i|/\gamma)) \} \,dt $$ which can then be split into a series of different integrals, and where the product is taken over all $i$. For full derivation details, see this answer in the context of an activation process.

If this "wave arrival time", $Y_i$, follows an Erlang distribution, the problem becomes slightly more complex. However, averaged simulations of the activation process under these distribution assumption say otherwise (that is, expression (1) is a good enough approximation). So there must be not much difference between considering the pure random variables, or its expected value. I would like to show this mathematically, either by deducing a similar expression if I take an Erlang-distributed random variable, or by other means. Any ideas?

Further insights: From this paper, we have the following result

Corollary 6.2. Let $Z = Y_1 + \cdots + Y_k$ be the sum of $k$ independent random variables having the Erlang distribution with parameter $(m_j, \gamma_j)$ for $j = 1, \dots, k$, and $\gamma_j \neq \gamma_p$ for $j \neq p$. Then the density for $Z$, $f_Z(t)$, for $t \geq 0$ is given by $$ f_Z(t) = \prod_{j=1}^k \gamma_j^{m_j} \left\{ \sum_{j=1}^k e^{-\gamma_j t} (-1)^{m_j-1} \times \sum_{\sum_{p=1}^k r_p = m_j-1, \, r_p \geq 0} \frac{(-t)^{r_j}}{r_j!} \prod_{q=1, q \neq j}^k \frac{(m_q + r_q - 1)!}{(\gamma_q - \gamma_j)^{m_q + r_q}} \right\}. $$ In our case, let $Z_i\equiv X_i+Y_i$ and set $k=2$, $(m_1,\gamma_1)=(1,\lambda)$, and $(m_2,\gamma_2)=(|i|,\gamma)$, the CDF $F_{Z_i}(t)$ is given by $$ F_{Z_i}(t) = 1 - e^{-\lambda t} + \lambda \gamma^{|i|} \sum_{r_2=0}^{|i|-1} \frac{r_1!}{(\lambda - \gamma)^{1 + r_1}} \left( \frac{\Gamma(r_2+1,0) - \Gamma(r_2+1, \gamma t)}{\gamma^{r_2+1}} \right) $$ where $\Gamma$ is the incomplete Gamma function, given by $$ \Gamma(s, z) = \int_z^\infty x^{s-1} e^{-x} \, dx, \quad \text{for } s > 0 \text{ and } z \geq 0. $$ From this question, we know that if the CDF of $Z_i$ is denoted by $F_{Z_i}(t)$, then the CDF of the minimum $Z$ is given by $F_Z(t)=1-[1-F_{Z_i}(t)]^{2n+1}$. Hence, $$ \begin{align} \mathbb{E}[Z]&=\int_0^\infty (1-F_Z(t))\,dt\\ &= \int_0^\infty \left( e^{-\lambda t} - \lambda \gamma^{|i|} \sum_{r_2=0}^{|i|-1} \frac{r_1!}{(\lambda - \gamma)^{1 + r_1}} \left( \frac{\Gamma(r_2+1, 0) - \Gamma(r_2+1, \gamma t)}{\gamma^{r_2+1}} \right) \right)^{2n+1} \, dt. \end{align} $$ I wonder if there is a simplification somewhere.

$\endgroup$
6
  • $\begingroup$ Have you looked at classical asymptotics of the min in the large n limit? The asymptotics are dictated by the shape of the tail of the pdf. $\endgroup$ Commented Oct 9, 2024 at 9:53
  • $\begingroup$ Is $Y_0$ always equal to zero? $\endgroup$ Commented Oct 10, 2024 at 4:29
  • $\begingroup$ The mean of $Z$ for $n=1$ seems to be $\frac{1}{9} \left(4 \left(\frac{1}{\gamma +2 \lambda }+\frac{1}{2 \gamma +\lambda }\right)+\frac{3}{\lambda }\right)$ if $\lambda \neq \gamma$. $\endgroup$ Commented Oct 11, 2024 at 6:22
  • $\begingroup$ The mean of $Z$ for $n=2$ seems to be $$\frac{13824 \gamma ^{11}+293760 \gamma ^{10} \lambda +2331360 \gamma ^9 \lambda ^2+9638920 \gamma ^8 \lambda ^3+23561620 \gamma ^7 \lambda ^4+36189166 \gamma ^6 \lambda ^5+35876495 \gamma ^5 \lambda ^6+23033010 \gamma ^4 \lambda ^7+9398420 \gamma ^3 \lambda ^8+2319480 \gamma ^2 \lambda ^9+311040 \gamma \lambda ^{10}+17280 \lambda ^{11}}{5 \lambda (4 \gamma +\lambda )^3 (3 \gamma +2 \lambda )^3 (2 \gamma +3 \lambda )^3 (\gamma +4 \lambda )^2}$$. $\endgroup$ Commented Oct 12, 2024 at 6:06
  • 1
    $\begingroup$ Symbolic means will likely get much more complicated with increasing values of $n$. But obtaining the means numerically for specific values of $n$, $\lambda$, and $\gamma$ appears doable. Are you still interested in this problem? $\endgroup$ Commented Oct 12, 2024 at 6:08

1 Answer 1

2
$\begingroup$

If $X_i\sim \text{Exponential}(\lambda)$ and $Y_i \sim \text{Erlang}(|i|,\gamma)$ for $i \neq 0$ and $\gamma>\lambda$, then the pdf of $Z_i=X_i+Y_i$ is given by

$$f_i(z)=\int_0^z \lambda e^{-\lambda x} \times \frac{\gamma ^{|i|} (z-x)^{|i|-1} e^{\gamma (x-z)}}{\Gamma (|i|)} dx=\frac{\lambda \gamma ^{|i|} e^{\lambda (-z)} (\gamma -\lambda )^{-|i|} (\Gamma (|i|)-\Gamma (|i|,z (\gamma -\lambda )))}{\Gamma (|i|)}$$

We have $f_{i}(z)=f_{-i}(z)$. The cdf for $Z_i$ is then

$$F_i (z)=\int_0^z f_i(t)dt=\frac{e^{\lambda (-z)} \left(\frac{\gamma }{\gamma -\lambda }\right)^{|i|} (\Gamma (|i|,z (\gamma -\lambda ))-\Gamma (|i|))-e^{\gamma (-z)} (\gamma z)^{|i|-1}-(|i|-1) \Gamma (|i|-1,z \gamma )+\Gamma (|i|)}{\Gamma (|i|)}$$

Define $Z=\min_{-n\leq i \leq n}\{Z_i\}$. The cdf for $Z$ is

$$\text{Pr}(Z\leq z)=1-(1-F_0(z)) \prod _{i=1}^n (1-F_i(z))^2$$

and the pdf for $Z$ is the derivative of the cdf with respect to $z$.

Using Mathematica we have the following starting with the cdf's for $Z_i$:

F[0] = 1 - E^(- λ z);
F[i_] := (-E^(-z γ) (z γ)^(-1 + i) + Gamma[i] - (-1 + i) Gamma[-1 + i, z γ] + 
  E^(-z λ) (γ/(γ - λ))^i (-Gamma[i] + Gamma[i, z (γ - λ)]))/Gamma[i]

This results in functions for the cdf, pdf, and mean of $Z$:

cdf[n_] := 1 - (1 - F[0]) Product[(1 - F[i])^2, {i, 1, n}]
pdf[n_] := FullSimplify[D[cdf[n], z], Assumptions -> {n ∈ PositiveIntegers, γ > λ > 0, z > 0}]

mean[n_] := Integrate[z  pdf[n], {z, 0, ∞}, 
  Assumptions ->{n ∈ PositiveIntegers, γ > λ > 0, z > 0}]

mean[1]

$$\frac{1}{9} \left(4 \left(\frac{1}{\gamma +2 \lambda }+\frac{1}{2 \gamma +\lambda }\right)+\frac{3}{\lambda }\right)$$

mean[2]

$$\frac{13824 \gamma ^{11}+293760 \gamma ^{10} \lambda +2331360 \gamma ^9 \lambda ^2+9638920 \gamma ^8 \lambda ^3+23561620 \gamma ^7 \lambda ^4+36189166 \gamma ^6 \lambda ^5+35876495 \gamma ^5 \lambda ^6+23033010 \gamma ^4 \lambda ^7+9398420 \gamma ^3 \lambda ^8+2319480 \gamma ^2 \lambda ^9+311040 \gamma \lambda ^{10}+17280 \lambda ^{11}}{5 \lambda (4 \gamma +\lambda )^3 (3 \gamma +2 \lambda )^3 (2 \gamma +3 \lambda )^3 (\gamma +4 \lambda )^2}$$

For $n > 2$ the symbolic results are likely too complicated and lengthy to be useful. But placing numeric values for $\lambda$ and $\gamma$ can produce nearly exact results for values of $n > 2$. For example, consider $n = 5$ and $\lambda = 1$ and $\gamma = 5$.

cdfZ = cdf[5] /. {λ -> 1, γ -> 5};
pdfZ = D[cdfZ, z];
m = NIntegrate[z  pdfZ, {z, 0, ∞}]
(* 0.321508 *)
$\endgroup$

Your Answer

By clicking “Post Your Answer”, you agree to our terms of service and acknowledge you have read our privacy policy.

Start asking to get answers

Find the answer to your question by asking.

Ask question

Explore related questions

See similar questions with these tags.